eating beets

This topic has expert replies
User avatar
Legendary Member
Posts: 1309
Joined: Wed Mar 17, 2010 11:41 pm
Thanked: 33 times
Followed by:5 members

eating beets

by pradeepkaushal9518 » Mon Apr 12, 2010 12:55 am
Eating beets significantly lowers the risk of cancer, according to an article in a nutritional magazine. The article refers to a study that found that people who consumed one or more beets per day were half as likely to be diagnosed with the disease as people who did not.
Which of the following, if true, most weakens the argument in the magazine article?
A) Another study found that people who consumed one tablespoon of flax seed oil per day were more than four times less likely to be diagnosed with cancer as those who did not.
B) Participants in the study reported consuming no vegetables other than beets.
C) The study was only conducted in one city.
D) In another experiment, cancer patients who ate one or more beets per day were no more likely to recover than those who ate no beets.
E) The participants in the study who ate beets were more likely to exercise regularly than those who did not eat beets

explain and answer

User avatar
Legendary Member
Posts: 1560
Joined: Tue Nov 17, 2009 2:38 am
Thanked: 137 times
Followed by:5 members

by thephoenix » Mon Apr 12, 2010 1:03 am
the conclusion is that Eating beets significantly lowers the risk of cancer

evidence is people who consumed one or more beets per day were half as likely to be diagnosed with the disease as people who did not.

so clearly author assumes that eating beets is the only reason for less likeliness of people being diagnosed with the disease

to weaken this assumption we need to prove that something else may be the reason
choice E does that
and therefore is correct
it states that they do excercise also

Junior | Next Rank: 30 Posts
Posts: 19
Joined: Tue Apr 06, 2010 1:07 am

by gmat.cracker24 » Mon Apr 12, 2010 1:19 am
IMO (E)

A:One tbsp of flax etc. is out of scope,we are concerned with beet only
B:This emphasizes that lowering of cancer is due to beet only amongts all veg
C:No. of cities is irrelevant
D:Its the likelyhood of cancer rather than its recuperative powers that is under consideration
E:It weakens as we are not sure its beets or the exercises that help reduces the risk

Junior | Next Rank: 30 Posts
Posts: 14
Joined: Thu Nov 19, 2009 6:08 pm

by ironstar37 » Mon Apr 12, 2010 12:03 pm
IMO (E). If the participants who ate beets also exercised regularly, this exercise could be the cause of lowering the risk of cancer.

(A) (B) (C) are out of scope.

(D) is referring to patients who already have cancer. Study is referring to the risk of getting cancer.

Master | Next Rank: 500 Posts
Posts: 182
Joined: Mon Apr 20, 2009 7:09 pm
Thanked: 1 times
Followed by:1 members

by akahuja143 » Mon Apr 12, 2010 12:09 pm
IMO E to weaken X - causes Y we just need to show that not X but Z causes Y, E does it

GMAT Instructor
Posts: 357
Joined: Wed Aug 12, 2009 8:31 pm
Thanked: 128 times
Followed by:7 members

by grockit_andrea » Mon Apr 12, 2010 12:29 pm
This is a classic causation/correlation argument: the conclusion is based on the faulty assumption that just because there is a correlation between X and Y, X must be causing Y. One of the best ways to weaken an argument like that is to show that there's some other outside factor that is in fact responsible for Y, and that happens to be correlated to X. Choice E does that.
Andrea A.
Grockit Tutor
https://www.grockit.com

Master | Next Rank: 500 Posts
Posts: 140
Joined: Fri Feb 05, 2010 2:43 pm
Thanked: 3 times
GMAT Score:720

by analyst218 » Tue Apr 13, 2010 1:09 pm
grockit_andrea wrote:This is a classic causation/correlation argument: the conclusion is based on the faulty assumption that just because there is a correlation between X and Y, X must be causing Y. One of the best ways to weaken an argument like that is to show that there's some other outside factor that is in fact responsible for Y, and that happens to be correlated to X. Choice E does that.
I got E too, but I have a question.
Couldn't this also be a sample size- generalization error?
the passage draws a conclusion abt risk for all ppl based on a study conducted
among sample of ppl, and if that pool of sample is skewed or biased (C ).
Can somebody explain why C is out of scope besides 'E is a better answer'?

User avatar
Newbie | Next Rank: 10 Posts
Posts: 2
Joined: Wed Jan 24, 2007 4:49 am

by mituag » Tue Apr 13, 2010 10:07 pm
analyst218 wrote:
I got E too, but I have a question.
Couldn't this also be a sample size- generalization error?
the passage draws a conclusion abt risk for all ppl based on a study conducted
among sample of ppl, and if that pool of sample is skewed or biased (C ).
Can somebody explain why C is out of scope besides 'E is a better answer'?
Lets look at C.

C) The study was only conducted in one city.

First question that should come to your mind is "so what?"
How does this weaken an argument that eating beets "lowers the risk of cancer"? It may be that for that one city this is true but in general it may not be. Looking at this we cannot surely say that this weakens the argument.

When it comes to "E", it provides and alternate reason which weakens the argument stating eating beets is not the only reason which reduces the risk of cancer.

Also I think C is attacking the premise which is wrong. E is successfully attacking the assumption.

Hope that helped.

Master | Next Rank: 500 Posts
Posts: 379
Joined: Wed Apr 07, 2010 12:53 am
Location: Chennai,India
Thanked: 3 times

by paddle_sweep » Sun Apr 18, 2010 1:28 am
"E" does not show any relationship with cancer. It does not say that exercising has any relationship with cancer. How can "E" be correct?

GMAT Instructor
Posts: 357
Joined: Wed Aug 12, 2009 8:31 pm
Thanked: 128 times
Followed by:7 members

by grockit_andrea » Sun Apr 18, 2010 5:18 am
The argument says that people who eat beets are less prone to developing cancer, so eating beets must prevent cancer. Choice E says that people who eat beets (at least the people who matter for our purpose: the ones in the study) are more likely to exercise. So maybe there's actually no relationship between beets and cancer. Maybe the relationship is between exercise and cancer, and eating beets just happens to be correlated to exercising. That's how choice E weakens.
Andrea A.
Grockit Tutor
https://www.grockit.com

User avatar
Master | Next Rank: 500 Posts
Posts: 212
Joined: Sat Dec 01, 2007 4:19 pm
Thanked: 5 times

by vscid » Sun Apr 18, 2010 6:39 am
paddle_sweep wrote:"E" does not show any relationship with cancer. It does not say that exercising has any relationship with cancer. How can "E" be correct?
It doesn't. But it raises doubts on the validity of the conclusion that states a relation between beet and cancer.
The GMAT is indeed adaptable. Whenever I answer RC, it proficiently 'adapts' itself to mark my 'right' answer 'wrong'.

Master | Next Rank: 500 Posts
Posts: 379
Joined: Wed Apr 07, 2010 12:53 am
Location: Chennai,India
Thanked: 3 times

by paddle_sweep » Sun Apr 18, 2010 8:21 pm
Thanks for the clarifications provided. Understood the reasoning behind the correct answer now.

Cheers

User avatar
Legendary Member
Posts: 1261
Joined: Sun Sep 14, 2008 3:46 am
Thanked: 27 times
GMAT Score:570

by reply2spg » Thu Apr 22, 2010 4:26 pm
Cause - Effect, E brings in other possibility. Therefore, E is correct